Do Lagrange's multipliers fail in this case?Lagrange multipliers - maximum and minimum values given constraintExplain Lagrange multipliers?Lagrange Multipliers to determine min and maxLagrange Multipliers: Find $min$ of $f(x,y)=3(x+1) +2(y-1)$ subject to the constraint $x^2+y^2=4$Finding minimum using Lagrange multipliersHow to use the Lagrange Multipliers to find the min and max of this function?Using Lagrange multipliers to find max and min values?Why am I allowed to assume x or y are equal to 0 in this Lagrange multipliers problem?Issues with the “standard” intuition behind Lagrange MultipliersMaximize a bivariate function under constraints by Lagrange multipliers

Rationale to prefer local variables over instance variables?

For which categories of spectra is there an explicit description of the fibrant objects via lifting properties?

Why does Solve lock up when trying to solve the quadratic equation with large integers?

Is it possible that a question has only two answers?

Signed and unsigned numbers

How does Ehrenfest's theorem apply to the quantum harmonic oscillator?

Does a difference of tense count as a difference of meaning in a minimal pair?

What is the generally accepted pronunciation of “topoi”?

Is a piano played in the same way as a harmonium?

Are small insurances worth it?

(Codewars) Linked Lists - Remove Duplicates

Would an aboleth's Phantasmal Force lair action be affected by Counterspell, Dispel Magic, and/or Slow?

MySQL importing CSV files really slow

Getting the || sign while using Kurier

Professor forcing me to attend a conference, I can't afford even with 50% funding

Outlet with 3 sets of wires

Why do phishing e-mails use faked e-mail addresses instead of the real one?

Which situations would cause a company to ground or recall a aircraft series?

Street obstacles in New Zealand

Finitely many repeated replacements

Doubts in understanding some concepts of potential energy

Proving a statement about real numbers

Recommendation letter by significant other if you worked with them professionally?

Having the player face themselves after the mid-game



Do Lagrange's multipliers fail in this case?


Lagrange multipliers - maximum and minimum values given constraintExplain Lagrange multipliers?Lagrange Multipliers to determine min and maxLagrange Multipliers: Find $min$ of $f(x,y)=3(x+1) +2(y-1)$ subject to the constraint $x^2+y^2=4$Finding minimum using Lagrange multipliersHow to use the Lagrange Multipliers to find the min and max of this function?Using Lagrange multipliers to find max and min values?Why am I allowed to assume x or y are equal to 0 in this Lagrange multipliers problem?Issues with the “standard” intuition behind Lagrange MultipliersMaximize a bivariate function under constraints by Lagrange multipliers













0












$begingroup$


Question:
Use Lagrange multipliers to find the maximum and minimum values of the function subject to the given constraint. (If an answer does not exist, enter DNE.)



f(x,y)= y^2 - x^2 and (1/4)x^2 + y^2 = 9



I've tried solving it like I would all other questions of this type, but I get λ=4 which doesn't seem right. Am I doing something wrong or is it a DNE situation?










share|cite|improve this question









New contributor




Sarah is a new contributor to this site. Take care in asking for clarification, commenting, and answering.
Check out our Code of Conduct.







$endgroup$







  • 1




    $begingroup$
    I get, when I solve the Lagrange equations, $x=0$ (and $lambda=1$) or $y=0$ (and $lambda=4$). Then there's a max at $x=0, y=pm 3$ and a min at $y=0, x=pm 6$.
    $endgroup$
    – ancientmathematician
    yesterday











  • $begingroup$
    @ancientmathematician Isn´t $lambda_2=colorred-4$ If we change $9$ to $9.01$ the minimum changes from $-36$ to $-36+(-4cdot 0.01)=-36.04$
    $endgroup$
    – callculus
    yesterday











  • $begingroup$
    @ancientmathematician Yes, I can comprehend that. But wouldn´t be then $lambda_1$ negative if $lambda_2=4$?
    $endgroup$
    – callculus
    yesterday











  • $begingroup$
    @callculus, you are right of course.Sorry.
    $endgroup$
    – ancientmathematician
    yesterday










  • $begingroup$
    @ancientmathematician There is no excuse needed. I was just wondering. Thanks for the replies.
    $endgroup$
    – callculus
    yesterday















0












$begingroup$


Question:
Use Lagrange multipliers to find the maximum and minimum values of the function subject to the given constraint. (If an answer does not exist, enter DNE.)



f(x,y)= y^2 - x^2 and (1/4)x^2 + y^2 = 9



I've tried solving it like I would all other questions of this type, but I get λ=4 which doesn't seem right. Am I doing something wrong or is it a DNE situation?










share|cite|improve this question









New contributor




Sarah is a new contributor to this site. Take care in asking for clarification, commenting, and answering.
Check out our Code of Conduct.







$endgroup$







  • 1




    $begingroup$
    I get, when I solve the Lagrange equations, $x=0$ (and $lambda=1$) or $y=0$ (and $lambda=4$). Then there's a max at $x=0, y=pm 3$ and a min at $y=0, x=pm 6$.
    $endgroup$
    – ancientmathematician
    yesterday











  • $begingroup$
    @ancientmathematician Isn´t $lambda_2=colorred-4$ If we change $9$ to $9.01$ the minimum changes from $-36$ to $-36+(-4cdot 0.01)=-36.04$
    $endgroup$
    – callculus
    yesterday











  • $begingroup$
    @ancientmathematician Yes, I can comprehend that. But wouldn´t be then $lambda_1$ negative if $lambda_2=4$?
    $endgroup$
    – callculus
    yesterday











  • $begingroup$
    @callculus, you are right of course.Sorry.
    $endgroup$
    – ancientmathematician
    yesterday










  • $begingroup$
    @ancientmathematician There is no excuse needed. I was just wondering. Thanks for the replies.
    $endgroup$
    – callculus
    yesterday













0












0








0





$begingroup$


Question:
Use Lagrange multipliers to find the maximum and minimum values of the function subject to the given constraint. (If an answer does not exist, enter DNE.)



f(x,y)= y^2 - x^2 and (1/4)x^2 + y^2 = 9



I've tried solving it like I would all other questions of this type, but I get λ=4 which doesn't seem right. Am I doing something wrong or is it a DNE situation?










share|cite|improve this question









New contributor




Sarah is a new contributor to this site. Take care in asking for clarification, commenting, and answering.
Check out our Code of Conduct.







$endgroup$




Question:
Use Lagrange multipliers to find the maximum and minimum values of the function subject to the given constraint. (If an answer does not exist, enter DNE.)



f(x,y)= y^2 - x^2 and (1/4)x^2 + y^2 = 9



I've tried solving it like I would all other questions of this type, but I get λ=4 which doesn't seem right. Am I doing something wrong or is it a DNE situation?







lagrange-multiplier






share|cite|improve this question









New contributor




Sarah is a new contributor to this site. Take care in asking for clarification, commenting, and answering.
Check out our Code of Conduct.











share|cite|improve this question









New contributor




Sarah is a new contributor to this site. Take care in asking for clarification, commenting, and answering.
Check out our Code of Conduct.









share|cite|improve this question




share|cite|improve this question








edited yesterday









Bernard

122k741116




122k741116






New contributor




Sarah is a new contributor to this site. Take care in asking for clarification, commenting, and answering.
Check out our Code of Conduct.









asked yesterday









SarahSarah

11




11




New contributor




Sarah is a new contributor to this site. Take care in asking for clarification, commenting, and answering.
Check out our Code of Conduct.





New contributor





Sarah is a new contributor to this site. Take care in asking for clarification, commenting, and answering.
Check out our Code of Conduct.






Sarah is a new contributor to this site. Take care in asking for clarification, commenting, and answering.
Check out our Code of Conduct.







  • 1




    $begingroup$
    I get, when I solve the Lagrange equations, $x=0$ (and $lambda=1$) or $y=0$ (and $lambda=4$). Then there's a max at $x=0, y=pm 3$ and a min at $y=0, x=pm 6$.
    $endgroup$
    – ancientmathematician
    yesterday











  • $begingroup$
    @ancientmathematician Isn´t $lambda_2=colorred-4$ If we change $9$ to $9.01$ the minimum changes from $-36$ to $-36+(-4cdot 0.01)=-36.04$
    $endgroup$
    – callculus
    yesterday











  • $begingroup$
    @ancientmathematician Yes, I can comprehend that. But wouldn´t be then $lambda_1$ negative if $lambda_2=4$?
    $endgroup$
    – callculus
    yesterday











  • $begingroup$
    @callculus, you are right of course.Sorry.
    $endgroup$
    – ancientmathematician
    yesterday










  • $begingroup$
    @ancientmathematician There is no excuse needed. I was just wondering. Thanks for the replies.
    $endgroup$
    – callculus
    yesterday












  • 1




    $begingroup$
    I get, when I solve the Lagrange equations, $x=0$ (and $lambda=1$) or $y=0$ (and $lambda=4$). Then there's a max at $x=0, y=pm 3$ and a min at $y=0, x=pm 6$.
    $endgroup$
    – ancientmathematician
    yesterday











  • $begingroup$
    @ancientmathematician Isn´t $lambda_2=colorred-4$ If we change $9$ to $9.01$ the minimum changes from $-36$ to $-36+(-4cdot 0.01)=-36.04$
    $endgroup$
    – callculus
    yesterday











  • $begingroup$
    @ancientmathematician Yes, I can comprehend that. But wouldn´t be then $lambda_1$ negative if $lambda_2=4$?
    $endgroup$
    – callculus
    yesterday











  • $begingroup$
    @callculus, you are right of course.Sorry.
    $endgroup$
    – ancientmathematician
    yesterday










  • $begingroup$
    @ancientmathematician There is no excuse needed. I was just wondering. Thanks for the replies.
    $endgroup$
    – callculus
    yesterday







1




1




$begingroup$
I get, when I solve the Lagrange equations, $x=0$ (and $lambda=1$) or $y=0$ (and $lambda=4$). Then there's a max at $x=0, y=pm 3$ and a min at $y=0, x=pm 6$.
$endgroup$
– ancientmathematician
yesterday





$begingroup$
I get, when I solve the Lagrange equations, $x=0$ (and $lambda=1$) or $y=0$ (and $lambda=4$). Then there's a max at $x=0, y=pm 3$ and a min at $y=0, x=pm 6$.
$endgroup$
– ancientmathematician
yesterday













$begingroup$
@ancientmathematician Isn´t $lambda_2=colorred-4$ If we change $9$ to $9.01$ the minimum changes from $-36$ to $-36+(-4cdot 0.01)=-36.04$
$endgroup$
– callculus
yesterday





$begingroup$
@ancientmathematician Isn´t $lambda_2=colorred-4$ If we change $9$ to $9.01$ the minimum changes from $-36$ to $-36+(-4cdot 0.01)=-36.04$
$endgroup$
– callculus
yesterday













$begingroup$
@ancientmathematician Yes, I can comprehend that. But wouldn´t be then $lambda_1$ negative if $lambda_2=4$?
$endgroup$
– callculus
yesterday





$begingroup$
@ancientmathematician Yes, I can comprehend that. But wouldn´t be then $lambda_1$ negative if $lambda_2=4$?
$endgroup$
– callculus
yesterday













$begingroup$
@callculus, you are right of course.Sorry.
$endgroup$
– ancientmathematician
yesterday




$begingroup$
@callculus, you are right of course.Sorry.
$endgroup$
– ancientmathematician
yesterday












$begingroup$
@ancientmathematician There is no excuse needed. I was just wondering. Thanks for the replies.
$endgroup$
– callculus
yesterday




$begingroup$
@ancientmathematician There is no excuse needed. I was just wondering. Thanks for the replies.
$endgroup$
– callculus
yesterday










1 Answer
1






active

oldest

votes


















0












$begingroup$

Hint: Use $$y^2=9-frac14x^2$$ and you will get a Problem in only one variable:$$f(x,pmsqrt9-frac14x^2)=9-frac14x^2-x^2$$






share|cite|improve this answer









$endgroup$












    Your Answer





    StackExchange.ifUsing("editor", function ()
    return StackExchange.using("mathjaxEditing", function ()
    StackExchange.MarkdownEditor.creationCallbacks.add(function (editor, postfix)
    StackExchange.mathjaxEditing.prepareWmdForMathJax(editor, postfix, [["$", "$"], ["\\(","\\)"]]);
    );
    );
    , "mathjax-editing");

    StackExchange.ready(function()
    var channelOptions =
    tags: "".split(" "),
    id: "69"
    ;
    initTagRenderer("".split(" "), "".split(" "), channelOptions);

    StackExchange.using("externalEditor", function()
    // Have to fire editor after snippets, if snippets enabled
    if (StackExchange.settings.snippets.snippetsEnabled)
    StackExchange.using("snippets", function()
    createEditor();
    );

    else
    createEditor();

    );

    function createEditor()
    StackExchange.prepareEditor(
    heartbeatType: 'answer',
    autoActivateHeartbeat: false,
    convertImagesToLinks: true,
    noModals: true,
    showLowRepImageUploadWarning: true,
    reputationToPostImages: 10,
    bindNavPrevention: true,
    postfix: "",
    imageUploader:
    brandingHtml: "Powered by u003ca class="icon-imgur-white" href="https://imgur.com/"u003eu003c/au003e",
    contentPolicyHtml: "User contributions licensed under u003ca href="https://creativecommons.org/licenses/by-sa/3.0/"u003ecc by-sa 3.0 with attribution requiredu003c/au003e u003ca href="https://stackoverflow.com/legal/content-policy"u003e(content policy)u003c/au003e",
    allowUrls: true
    ,
    noCode: true, onDemand: true,
    discardSelector: ".discard-answer"
    ,immediatelyShowMarkdownHelp:true
    );



    );






    Sarah is a new contributor. Be nice, and check out our Code of Conduct.









    draft saved

    draft discarded


















    StackExchange.ready(
    function ()
    StackExchange.openid.initPostLogin('.new-post-login', 'https%3a%2f%2fmath.stackexchange.com%2fquestions%2f3140989%2fdo-lagranges-multipliers-fail-in-this-case%23new-answer', 'question_page');

    );

    Post as a guest















    Required, but never shown

























    1 Answer
    1






    active

    oldest

    votes








    1 Answer
    1






    active

    oldest

    votes









    active

    oldest

    votes






    active

    oldest

    votes









    0












    $begingroup$

    Hint: Use $$y^2=9-frac14x^2$$ and you will get a Problem in only one variable:$$f(x,pmsqrt9-frac14x^2)=9-frac14x^2-x^2$$






    share|cite|improve this answer









    $endgroup$

















      0












      $begingroup$

      Hint: Use $$y^2=9-frac14x^2$$ and you will get a Problem in only one variable:$$f(x,pmsqrt9-frac14x^2)=9-frac14x^2-x^2$$






      share|cite|improve this answer









      $endgroup$















        0












        0








        0





        $begingroup$

        Hint: Use $$y^2=9-frac14x^2$$ and you will get a Problem in only one variable:$$f(x,pmsqrt9-frac14x^2)=9-frac14x^2-x^2$$






        share|cite|improve this answer









        $endgroup$



        Hint: Use $$y^2=9-frac14x^2$$ and you will get a Problem in only one variable:$$f(x,pmsqrt9-frac14x^2)=9-frac14x^2-x^2$$







        share|cite|improve this answer












        share|cite|improve this answer



        share|cite|improve this answer










        answered yesterday









        Dr. Sonnhard GraubnerDr. Sonnhard Graubner

        77.4k42866




        77.4k42866




















            Sarah is a new contributor. Be nice, and check out our Code of Conduct.









            draft saved

            draft discarded


















            Sarah is a new contributor. Be nice, and check out our Code of Conduct.












            Sarah is a new contributor. Be nice, and check out our Code of Conduct.











            Sarah is a new contributor. Be nice, and check out our Code of Conduct.














            Thanks for contributing an answer to Mathematics Stack Exchange!


            • Please be sure to answer the question. Provide details and share your research!

            But avoid


            • Asking for help, clarification, or responding to other answers.

            • Making statements based on opinion; back them up with references or personal experience.

            Use MathJax to format equations. MathJax reference.


            To learn more, see our tips on writing great answers.




            draft saved


            draft discarded














            StackExchange.ready(
            function ()
            StackExchange.openid.initPostLogin('.new-post-login', 'https%3a%2f%2fmath.stackexchange.com%2fquestions%2f3140989%2fdo-lagranges-multipliers-fail-in-this-case%23new-answer', 'question_page');

            );

            Post as a guest















            Required, but never shown





















































            Required, but never shown














            Required, but never shown












            Required, but never shown







            Required, but never shown

































            Required, but never shown














            Required, but never shown












            Required, but never shown







            Required, but never shown







            Popular posts from this blog

            Lowndes Grove History Architecture References Navigation menu32°48′6″N 79°57′58″W / 32.80167°N 79.96611°W / 32.80167; -79.9661132°48′6″N 79°57′58″W / 32.80167°N 79.96611°W / 32.80167; -79.9661178002500"National Register Information System"Historic houses of South Carolina"Lowndes Grove""+32° 48' 6.00", −79° 57' 58.00""Lowndes Grove, Charleston County (260 St. Margaret St., Charleston)""Lowndes Grove"The Charleston ExpositionIt Happened in South Carolina"Lowndes Grove (House), Saint Margaret Street & Sixth Avenue, Charleston, Charleston County, SC(Photographs)"Plantations of the Carolina Low Countrye

            random experiment with two different functions on unit interval Announcing the arrival of Valued Associate #679: Cesar Manara Planned maintenance scheduled April 23, 2019 at 00:00UTC (8:00pm US/Eastern)Random variable and probability space notionsRandom Walk with EdgesFinding functions where the increase over a random interval is Poisson distributedNumber of days until dayCan an observed event in fact be of zero probability?Unit random processmodels of coins and uniform distributionHow to get the number of successes given $n$ trials , probability $P$ and a random variable $X$Absorbing Markov chain in a computer. Is “almost every” turned into always convergence in computer executions?Stopped random walk is not uniformly integrable

            How should I support this large drywall patch? Planned maintenance scheduled April 23, 2019 at 00:00UTC (8:00pm US/Eastern) Announcing the arrival of Valued Associate #679: Cesar Manara Unicorn Meta Zoo #1: Why another podcast?How do I cover large gaps in drywall?How do I keep drywall around a patch from crumbling?Can I glue a second layer of drywall?How to patch long strip on drywall?Large drywall patch: how to avoid bulging seams?Drywall Mesh Patch vs. Bulge? To remove or not to remove?How to fix this drywall job?Prep drywall before backsplashWhat's the best way to fix this horrible drywall patch job?Drywall patching using 3M Patch Plus Primer